1answer.
Ask question
Login Signup
Ask question
All categories
  • English
  • Mathematics
  • Social Studies
  • Business
  • History
  • Health
  • Geography
  • Biology
  • Physics
  • Chemistry
  • Computers and Technology
  • Arts
  • World Languages
  • Spanish
  • French
  • German
  • Advanced Placement (AP)
  • SAT
  • Medicine
  • Law
  • Engineering
sdas [7]
3 years ago
9

Scientific notation for 1,360

Mathematics
2 answers:
VARVARA [1.3K]3 years ago
5 0

We have a large numer and using scientfic notation is a great way to break down the number. So let's break it down!

1,000 x 1 = 1,000

3 x 100 = 300

6 x 10 = 60

3 x 1 = 3

So the form for scientific notation is a x 10b and we need to convert it into decial form.

1,360 becomes 1.36 (forget the 0) for a. 10 b is 103

1.36 x 103. Notation will always have a square. Our b.

10^3

SO, now we have

1.36 x 10^3 = 1,360




emmainna [20.7K]3 years ago
4 0

The scientific notation:

a\cdot10^k where 1\leq a

We have:

1,360=1\underbrace{.360}_{\leftarrow3}\cdot10^3=\boxed{1.36\cdot10^3}

You might be interested in
Question 1 of 10
Lorico [155]
It’s should be -10 answer
8 0
3 years ago
WHICH IS A TRUE STATEMENT BASED ON THE DOT PLOTS BELOW 10 POINTS
Nata [24]

Answer:

C. Set B has the greater range.

Step-by-step explanation:

We can start out with listing out each set of data in a way that makes it easier to work with.

Set A:

20,20,30,30,40,40,40,40,40,50,50,60,60,60

Set B:

10,10,20,20,20,30,30,30,30,40,50,50,60

We can start by trying to eliminate answer options; starting with D.

To get the mean of a set of data, we need to add all numbers in the data set and then divide by the number of dots on the plot.

For Set B: 10+10+20+20+20+20+30+30+30+30+40+50+50+60=420 then divide by number of dots (14) 420/14=30 The mean of Set B is equal to 30.

Solve the mean for Set A to compare. The mean is 41. D is not true so we can eliminate this answer choice.

We can look at the median of both sets so that we can eliminate answer choice B. The median is the middle most point in the set of data.

Set A median: 40

Set B median: 30

We can eliminate B because it does not have the greater median.

On to option C: range.

To find the range of each data set, we subtract the smallest number from the biggest number.

Set A: 60-20=40

Set B: 60-10= 50

Set B does in fact have a greater range than Set A. Answer choice C is the correct option.

6 0
3 years ago
How would I show 2/5 as a division problem? Please explain using complete sentences.
dangina [55]

Answer:

There are many ways to show it like-

Step-by-step explanation:

2/5 as it’s already shown, or you can use 2 divide by 5, many people use 2[5] that’s the ways they showed my in my school idk if its different for you

5 0
3 years ago
Read 2 more answers
A bus travels 36 miles in 45 minutes. How many miles will the bus travel in 60 minutes at this rate. Show a ratio.
ankoles [38]
A rate is a special ratio in which the two terms are in different units. A rate is a little bit different than the ratio, it is a special ratio. For this case, the rate would have units of miles per minutes. We calculate as follows:

rate = 36 miles / 45 min = 0.8 miles per min
distance traveled = 0.8 miles per min ( 60 min ) = 48 miles
3 0
3 years ago
Help pls just help i can't take this anymore​
Romashka-Z-Leto [24]
It is C
Ksixkskxksmxmamlsldllxlallzlskmclaldlldle
4 0
3 years ago
Read 2 more answers
Other questions:
  • Yana took a 15-hour flight to Korea he slept for 1/3 of the flight how many minutes did he sleep
    8·1 answer
  • Can someone explain Part B to me please? I'm trying to practice for SBAC but I don't get this.
    12·1 answer
  • About how accurately must the tank's exterior diameter be measured to calculate the amount of paint it will take to paint the si
    15·1 answer
  • What is the answer? A 1. B. -2 C. 1 , 2 D. 1, -1
    10·1 answer
  • Please Help Me. Add (2x^2+4−x^3)+(−4x^2+6x^3−2) Express the answer in standard form. Enter your answer in the box.
    10·1 answer
  • 3. Kevin and Hannah are both saving money for a new television. Kevin has already saved $ 47 and
    11·1 answer
  • 14 + (-26) - 14 -26 <br> pls explain how to get the answer
    12·2 answers
  • Question 8 of 10
    6·1 answer
  • 96 words typed in 3 minutes; 160 words typed in 5 minutes
    10·1 answer
  • Last year scott ate 84 cups of ice cream.how many quarts did he eat?
    13·1 answer
Add answer
Login
Not registered? Fast signup
Signup
Login Signup
Ask question!